Limit of nth root of n factorial

Поділитися
Вставка
  • Опубліковано 22 жов 2024

КОМЕНТАРІ • 8

  • @aeugh4200
    @aeugh4200 2 дні тому +4

    i used the stirling approximation and for large n it converges to (n/e), so the limit (n->inf) -> inf

  • @peco1738
    @peco1738 2 дні тому +2

    nice solution bro

  • @AaryanJain-lk3jt
    @AaryanJain-lk3jt 2 дні тому +2

    i got infinity raised to 0, as 1/n approached 0 and n! approached infinity, which would then be 1, right?

    • @AaryanJain-lk3jt
      @AaryanJain-lk3jt 2 дні тому

      but then you can reason that n! as n approaches infinity is infinity multiplied to itself an infinite amount of times, or infinity^infinity, and (infinity^infinity)^(1/infinity) = infinity^1 = infinity

    • @onelemmaatatime
      @onelemmaatatime  2 дні тому

      Yes things get tricky when one tries to do arithmetic with infinities; not all of the usual rules apply. This is part of the reason for introducing limits.

    • @isavenewspapers8890
      @isavenewspapers8890 2 дні тому +1

      ∞^0 is one of the indeterminate forms. An indeterminate form is a form an expression can take when evaluating a limit. If you get an indeterminate form, it means that you need to do more work to determine the limit.
      In the case of ∞^0, it's possible that the infinity represents a growing expression, while the 0 represents an expression approaching 0. When this happens, it's possible for the ∞ part to "overpower" the 0 part. One example is the case shown in the video.
      A simpler example is the limit of (n^n)^(1 / n) as n approaches infinity. If you just plug in n = ∞ and work things out, you'll end up with ∞^0. But you can instead simplify the original expression first:
      (n^n)^(1 / n)
      = n^(n * 1 / n)
      = n^(n / n)
      = n^1
      = n.
      Now you can take the limit as n approaches infinity. As you can see, the limit is infinity, not 1.

  • @aeugh4200
    @aeugh4200 2 дні тому

    [edit] The other limit of (n!)^(1/n) where n -> 0:
    exp((ln(x!)/x) -> numerator and denominator are 0
    l'Hopital's Rule
    d/dx ln(x!))/x = digamma(x+1) * gamma(x+1) / 1
    plug in x and the solution in e^digamma(1) = e^(-gamma) =~ 0.56146